[obm-l] resolução - olimpíada cearense

2004-04-27 Por tôpico Max




Claúdio, 
ONicolau resolveu e discutiu aquestão proposta na olimpíada 
cearense. Está resolvida no link abaixo, está resolvida em três etapas.
http://www.mat.puc-rio.br/~nicolau/olimp/obm-l.200401/msg00368.html


Re: [obm-l] COMBINATÓRIA

2004-04-27 Por tôpico Augusto Cesar de Oliveira Morgado



Imagine que as faces sejam diferentes; por exemplo, imagine as faces numeradas. Ha 6!=720 modos de pinta-las. Com a condição SE UMA MANEIRA É CONSIDERADA IDÊNTICA A OUTRA, DESDE QUE POSSA SER OBTIDA A PARTIR DESTA POR ROTAÇÃO DO CUBO, perceba que um cubo pode ser rodado de 24 modos (pois há 6 modos de escolher a face de baixo e 4 modos de escolher nessa face a aresta que fica de frente) e, agora, esses 24 cubos sao iguais. A resposta eh 720/24 = 30.

== 
Mensagem  enviada  pelo  CIP  WebMAIL  - Nova Geração - v. 2.1 
CentroIn Internet Provider          http://www.centroin.com.br 
Tel: (21) 2542-4849, (21) 2295-3331        Fax: (21) 2295-2978 
Empresa 100% Brasileira - Desde 1992 prestando servicos online 


-- Original Message ---
From: [EMAIL PROTECTED] 
To: [EMAIL PROTECTED] 
Sent: Mon, 26 Apr 2004 00:11:53 EDT 
Subject: [obm-l] COMBINATÓRIA 

 DISPOMOS DE SEIS CORES DIFERENTES. CADA FACE DE UM CUBO SERÁ PINTADA COM UMA COR DIFERENTE, DE FORMA QUE AS SEIS CORES SEJAM UTILIZADAS. DE QUANTAS MANEIRAS DIFERENTES ISSO PODE SER FEITO, SE UMA MANEIRA É CONSIDEADA IDÊNTICA A OUTRA, DESDE QUE POSSA SER OBTIDA A PARTIR DESTA POR ROTAÇÃO DO CUBO.
 
 Ps¹: peço muita paciência na hora da explicação
 Ps²: a resposta do problema é 30
 
 Grato, Junior 
--- End of Original Message ---






Re: [obm-l] Simetria de matrizes

2004-04-27 Por tôpico Alan Pellejero
Como se prova se é verdadeira ou falsa??
Artur Costa Steiner [EMAIL PROTECTED] wrote:
Ola pessoal...  To com um pouco de dificuldade pra provar a seguinte questao:  "O produto de duas matrizes simetricas e necessariamente simetrico? Prove sua resposta."Isto eh falso. O que h verdade eh que (AB)' = BA, onde' signfica a transposta.Artur__Do you Yahoo!?Yahoo! Photos: High-quality 4x6 digital prints for 25¢http://photos.yahoo.com/ph/print_splash=Instruções para entrar na lista, sair da lista e usar a lista emhttp://www.mat.puc-rio.br/~nicolau/olimp/obm-l.html=Yahoo! Messenger - Fale com seus amigos online. Instale agora!

RES: [obm-l] Simetria de matrizes

2004-04-27 Por tôpico Cloves Jr



eh exatamente essa a 
minha duvida... eu cheguei exatamente onde vc chegou mas tb naum consigo 
concluir...

eu tb achei um 
contra-exemplo que prova que naum eh simetrica mas tb queira saber num caso 
geral...

Cloves

  -Mensagem original-De: [EMAIL PROTECTED] 
  [mailto:[EMAIL PROTECTED]Em nome de Alan 
  PellejeroEnviada em: terça-feira, 27 de abril de 2004 
  09:37Para: [EMAIL PROTECTED]Assunto: Re: [obm-l] 
  Simetria de matrizes
  Como se prova se é verdadeira ou falsa??
  Artur Costa Steiner [EMAIL PROTECTED] 
  wrote:
  Ola 
pessoal...  To com um pouco de dificuldade pra provar a 
seguinte questao:  "O produto de duas matrizes 
simetricas e necessariamente simetrico? Prove sua 
resposta."Isto eh falso. O que h verdade eh que (AB)' = BA, 
onde' signfica a 
transposta.Artur__Do 
you Yahoo!?Yahoo! Photos: High-quality 4x6 digital prints for 
25¢http://photos.yahoo.com/ph/print_splash=Instruções 
para entrar na lista, sair da lista e usar a lista 
emhttp://www.mat.puc-rio.br/~nicolau/olimp/obm-l.html=
  
  
  Yahoo! 
  Messenger - Fale com seus amigos online. Instale 
  agora!


Re: [obm-l] Simetria de matrizes

2004-04-27 Por tôpico Artur Costa Steiner
Como se prova se é verdadeira ou falsa??. 

Para provar que uma afirmacao eh verdadeira, voce tem que recorrer a
racicinio logico. Mas no caso naum eh possivel, a menos que a matriz tenha
dimensao =2. Basta dar um contra exemplo Considere  

  2 1   5   
  1 0   9   e
  5 9   13  

 1  2   -1
 2  5   -8
-1  -8 2

Ambas sao simetricas e seu produto eh 

-1  -31 0
-8  -70 17
10  -49  -51  , que naum eh simetrica


Artur






OPEN Internet
@ Primeiro provedor do DF com anti-vírus no servidor de e-mails @


=
Instruções para entrar na lista, sair da lista e usar a lista em
http://www.mat.puc-rio.br/~nicolau/olimp/obm-l.html
=


[obm-l] p(n+1) - p(n)

2004-04-27 Por tôpico Cláudio \(Prática\)
O que se sabe sobre a seqûencia d(n) = p(n+1) - p(n), onde p(n) = n-ésimo
primo?

O problema abaixo mostra que limsup d(n) = +infinito.

Existe alguma cota inferior conhecida para liminf d(n)?

[]s,
Claudio.

- Original Message -
From: [EMAIL PROTECTED]
To: [EMAIL PROTECTED]
Sent: Saturday, April 24, 2004 10:55 PM
Subject: Re: [obm-l] DUVIDA - Primo


  Alguem pode me dar uma ajuda nesta questão:
 
  Seja p(n) o n-ésimo número primo ( p(1) = 2, p(2) = 3, p(3) = 5 ...).
  Demonstrar que o conjunto formado pelas diferenças p(n + 1) - p(n)
  possui um numero infinito de elementos.
  [...]

 Note que isto equivale a provar que o conjunto das diferenças p(n+1)-p(n)
 contém números arbitrariamente grandes, i.e. para todo N natural, existem
 N naturais compostos consecutivos.

 []s,

 --
 Fábio ctg \pi Dias Moreira


 =
 Instruções para entrar na lista, sair da lista e usar a lista em
 http://www.mat.puc-rio.br/~nicolau/olimp/obm-l.html
 =

=
Instruções para entrar na lista, sair da lista e usar a lista em
http://www.mat.puc-rio.br/~nicolau/olimp/obm-l.html
=


Re: [obm-l] p(n+1) - p(n)

2004-04-27 Por tôpico Johann Peter Gustav Lejeune Dirichlet
Bem, o livro do Gugu e do Saldanha, comm o titulo grande, fala um pouco disso...

http://www.mat.puc-rio.br/~nicolau/papers/mersenne/mersenne.htmlCláudio_(Prática) [EMAIL PROTECTED] wrote:
O que se sabe sobre a seqûencia d(n) = p(n+1) - p(n), onde p(n) = n-ésimoprimo?O problema abaixo mostra que limsup d(n) = +infinito.Existe alguma cota inferior conhecida para liminf d(n)?[]s,Claudio.- Original Message -From: <[EMAIL PROTECTED]>To: <[EMAIL PROTECTED]>Sent: Saturday, April 24, 22004 10:55 PMSubject: Re: [obm-l] DUVIDA - Primo  Alguem pode me dar uma ajuda nesta questão:   Seja p(n) o n-ésimo número primo ( p(1) = 2, p(2) = 3, p(3) = 5 ...).  Demonstrar que o conjunto formado pelas diferenças p(n + 1) - p(n)  possui um numero infinito de elementos.  [...] Note que isto equivale a provar que o conjunto das diferenças p(n+1)-p(n) contém números arbitrariamente grandes, i.e. para todo N natural,
 existem N naturais compostos consecutivos. []s, -- Fábio "ctg \pi" Dias Moreira == Instruções para entrar na lista, sair da lista e usar a lista em http://www.mat.puc-rio.br/~nicolau/olimp/obm-l.html ==Instruções para entrar na lista, sair da lista e usar a lista emhttp://www.mat.puc-rio.br/~nicolau/olimp/obm-l.html=

TRANSIRE SVVM PECTVS MVNDOQVE POTIRI
CONGREGATI EX TOTO ORBE MATHEMATICI OB SCRIPTA INSIGNIA TRIBVERE
Fields Medal(John Charles Fields)Yahoo! Messenger - Fale com seus amigos online. Instale agora!

Re: [obm-l] p(n+1) - p(n)

2004-04-27 Por tôpico Johann Peter Gustav Lejeune Dirichlet
Bem, o livro do Gugu e do Saldanha, comm o titulo grande, fala um pouco disso...

http://www.mat.puc-rio.br/~nicolau/papers/mersenne/mersenne.htmlCláudio_(Prática) [EMAIL PROTECTED] wrote:
O que se sabe sobre a seqûencia d(n) = p(n+1) - p(n), onde p(n) = n-ésimoprimo?O problema abaixo mostra que limsup d(n) = +infinito.Existe alguma cota inferior conhecida para liminf d(n)?[]s,Claudio.- Original Message -From: <[EMAIL PROTECTED]>To: <[EMAIL PROTECTED]>Sent: Saturday, April 24, 22004 10:55 PMSubject: Re: [obm-l] DUVIDA - Primo  Alguem pode me dar uma ajuda nesta questão:   Seja p(n) o n-ésimo número primo ( p(1) = 2, p(2) = 3, p(3) = 5 ...).  Demonstrar que o conjunto formado pelas diferenças p(n + 1) - p(n)  possui um numero infinito de elementos.  [...] Note que isto equivale a provar que o conjunto das diferenças p(n+1)-p(n) contém números arbitrariamente grandes, i.e. para todo N natural,
 existem N naturais compostos consecutivos. []s, -- Fábio "ctg \pi" Dias Moreira == Instruções para entrar na lista, sair da lista e usar a lista em http://www.mat.puc-rio.br/~nicolau/olimp/obm-l.html ==Instruções para entrar na lista, sair da lista e usar a lista emhttp://www.mat.puc-rio.br/~nicolau/olimp/obm-l.html=

TRANSIRE SVVM PECTVS MVNDOQVE POTIRI
CONGREGATI EX TOTO ORBE MATHEMATICI OB SCRIPTA INSIGNIA TRIBVERE
Fields Medal(John Charles Fields)Yahoo! Messenger - Fale com seus amigos online. Instale agora!

[obm-l] DUVIDA - funçao

2004-04-27 Por tôpico João Silva
Outra dúvida:

- Uma função f : A -- B (em que A é o conjunto dos numeros reais positivos não - nulos e B o conjunto dos reais)é estritamente crescentte e para "x" e "y" pertencentes a A temos: f (x.y) = f(x) + f(y) . Sabe-se ainda que f(1) = 0 e f(2) = 1. Demonstrar que f(3) é irracional.Yahoo! Messenger - Fale com seus amigos online. Instale agora!

Re: [obm-l] DUVIDA - funçao

2004-04-27 Por tôpico Ricardo Bittencourt
João Silva wrote:
- Uma função f : A -- B (em que A é o conjunto dos numeros reais 
positivos não - nulos e B o conjunto dos reais) é estritamente crescente 
e para x e y pertencentes a A temos: f (x.y) = f(x) + f(y) . Sabe-se 
ainda que f(1) = 0 e f(2) = 1. Demonstrar que f(3) é irracional.
f(sqrt(2)*sqrt(2))=2*f(sqrt(2))=f(2)=1
logo f(sqrt(2))=1/2
Daí fica claro que uma função f é o log na base 2 né? Pois:
log2 (1) = 0
log2 (sqrt(2))=1/2
log2 (2) = 1
log2 (ab) = log2 (a) + log2 (b)
Então resta provar que log2(3) é irracional.
Pra isso acontecer, 3=2^(p/q) com p,q inteiros.
Mas então 3^q=2^p, e com p,q inteiros isso é impossível.
Hum.. resta provar que log2 é a única função f que
satisfaz o enunciado, isso eu não sei fazer.

Ricardo Bittencourt   http://www.mundobizarro.tk
[EMAIL PROTECTED]   tenki ga ii kara sanpo shimashou
-- União contra o forward - crie suas proprias piadas --
=
Instruções para entrar na lista, sair da lista e usar a lista em
http://www.mat.puc-rio.br/~nicolau/olimp/obm-l.html
=


Re: [obm-l] DUVIDA - funçao

2004-04-27 Por tôpico Ricardo Bittencourt
Ricardo Bittencourt wrote:
Então resta provar que log2(3) é irracional.
Pra isso acontecer, 3=2^(p/q) com p,q inteiros.
Mas então 3^q=2^p, e com p,q inteiros isso é impossível.
Ahn, eu perdi a medalha de ouro na obm de 93 por
falta de rigor, pelo jeito não aprendi nada de lá pra cá hehe.
deixando mais rigoroso então:
Pra isso acontecer, 3=2^(p/q) com p,q inteiros, e q!=0.
Mas então 3^q=2^p, e com p,q inteiros a única solução
é p=q=0, mas como q!=0 nenhuma solução é válida.

Ricardo Bittencourt   http://www.mundobizarro.tk
[EMAIL PROTECTED]   tenki ga ii kara sanpo shimashou
-- União contra o forward - crie suas proprias piadas --
=
Instruções para entrar na lista, sair da lista e usar a lista em
http://www.mat.puc-rio.br/~nicolau/olimp/obm-l.html
=


[obm-l] Re: [obm-l] DUVIDA - funçao

2004-04-27 Por tôpico Cláudio \(Prática\)



Outra dúvida:

- Uma função f : A -- B (em que A é o conjunto dos numeros reais 
positivos não - nulos e B o conjunto dos reais)é estritamente crescente e 
para "x" e "y" pertencentes a A temos: f (x.y) = f(x) + f(y) . Sabe-se ainda que 
f(1) = 0 e f(2) = 1. Demonstrar que f(3) é irracional.

É fácilprovar (por exemplo, por indução) que, 
para n inteiro positivo, vale f(x^n) = n*f(x).
Em particular, f(2^n) = n*f(2) = n.
Como f é estritamente crescente, f é 
injetiva.
Logo, f(x) é inteiro positivo == x = 2^n, 
com n inteiro positivo.

Suponhamos que f(3) seja racional, ou seja, f(3) = 
p/q com p,q inteiros positivos primos entre si.
(podemos supor que p e q são ambos positivos porque 
f(3)  f(2)  0).

f(3^q) = q*f(3) = q*(p/q) = p = inteiro positivo 
==
3^q = 2^n para algum inteiro n ==
contradição ao teorema fundamental da aritmética 
==
f(3) é irracional

[]s,
Claudio.


Re: [obm-l] p(n+1) - p(n)

2004-04-27 Por tôpico Cláudio \(Prática\)



Muito obrigado! 
Eu até conhecia essa referência mas nunca tinha 
lido essa parte em particular.

[]s,
Claudio.


  - Original Message - 
  From: 
  Johann Peter Gustav Lejeune 
  Dirichlet 
  To: [EMAIL PROTECTED] 
  Sent: Tuesday, April 27, 2004 12:49 
  PM
  Subject: Re: [obm-l] p(n+1) - p(n)
  
  Bem, o livro do Gugu e do Saldanha, comm o titulo grande, fala um pouco 
  disso...
  
  http://www.mat.puc-rio.br/~nicolau/papers/mersenne/mersenne.htmlCláudio_(Prática) 
  [EMAIL PROTECTED] 
  wrote:
  O 
que se sabe sobre a seqûencia d(n) = p(n+1) - p(n), onde p(n) = 
n-ésimoprimo?O problema abaixo mostra que limsup d(n) = 
+infinito.Existe alguma cota inferior conhecida para liminf 
d(n)?[]s,Claudio.- Original Message -From: 
<[EMAIL PROTECTED]>To: <[EMAIL PROTECTED]>Sent: Saturday, 
April 24, 2004 10:55 PMSubject: Re: [obm-l] DUVIDA - 
Primo  Alguem pode me dar uma ajuda nesta 
questão:   Seja p(n) o n-ésimo número primo ( p(1) = 
2, p(2) = 3, p(3) = 5 ...).  Demonstrar que o conjunto formado 
pelas diferenças p(n + 1) - p(n)  possui um numero infinito de 
elementos.  [...] Note que isto equivale a 
provar que o conjunto das diferenças p(n+1)-p(n) contém números 
arbitrariamente grandes, i.e. para todo N nat! ural, existem N 
naturais compostos consecutivos. []s, 
-- Fábio "ctg \pi" Dias Moreira 
= 
Instruções para entrar na lista, sair da lista e usar a lista em 
http://www.mat.puc-rio.br/~nicolau/olimp/obm-l.html 
==Instruções 
para entrar na lista, sair da lista e usar a lista 
emhttp://www.mat.puc-rio.br/~nicolau/olimp/obm-l.html=
  
  
  TRANSIRE SVVM PECTVS MVNDOQVE 
  POTIRI
  CONGREGATI EX TOTO ORBE MATHEMATICI OB 
  SCRIPTA INSIGNIA TRIBVERE
  Fields Medal(John Charles 
  Fields)
  
  
  Yahoo! 
  Messenger - Fale com seus amigos online. Instale 
  agora!


[obm-l] RE: [obm-l] Re: [obm-l]_Feiticeira_de_Gauss,_Geometria_hiperbólica_e_Riemanianna

2004-04-27 Por tôpico Leandro Lacorte Recova








Alan, 



Sobre a sua primeira
pergunta, nao sei de onde voce tirou isso ! 



Se quiser ler sobre
geometria riemaniana va ao www.google.com e
procure algo por la. 



Ou se quiser ler algo e
ja tem um background de geometria diferencial classica , calculo avancado,
algebra linear, pode pegar o livro do Manfredo Geometria Riemaniana
e vera o que isso significa. 





Leandro. 



-Original Message-
From: [EMAIL PROTECTED]
[mailto:[EMAIL PROTECTED] On Behalf
Of Alan Pellejero
Sent: Monday, April 26, 2004 1:42
PM
To: [EMAIL PROTECTED]
Subject: Re: [obm-l] Re:
[obm-l]_Feiticeira_de_Gauss,_Geometria_hiperbólica_e_Riemanianna






Por acaso a riemanianna é projetada numa circunferência??





E, na teoria da relatividade, que eu já andei olhando alguma coisa, por
que a massa varia???





Obrigado





Alan Pellejero
Nicolau C.
Saldanha [EMAIL PROTECTED] wrote:





On Mon, Apr 26, 2004 at 12:53:00PM -0300, Alan Pellejero wrote:
 Pessoal, desculpem-me se este assunto for off-topic, na verdade eu nem sei
o
 que não é off-topic, mas eu gostaria de saber o que é a feiticeira de
gauss.

Acho que a reclamação do Claudio não foi no sentido de que a sua mensagem
fosse off-topic, foi no sentido de dizer que a pergunta já tinha sido
respondida.

As perguntas abaixo não são off-topic mas exigem livros inteiros
para responder. Vou dar respostas sumárias.

 Gostaria de saber também o seguinte:
 
 se a geometria euclidiana é projetada no plano, onde é projetada a
 riemanianna e a hiperbólica???

Esta pergunta não faz sentido.

 Ouvi tambeém comentários sobre uma outra geometria que fazia a projeção
num
 plano que parece uma cela de cavalo...

O plano hipe! rbólico tem curvatura negativa; a sela de cavalo também tem;
as semelhanças param mais ou menos por aí.

 Por que tantas geomeetrias???

Por que são úteis. Existem tantas geometrias quanto os mátemáticos são
capazes de inventar e as mais úteis ficam famosas.

 Existem outras além dessas?

Muitas.

 Por que Einstein precisou da riemanianna pra formular a teoria da
 relatividade???

Acho que para responder esta pergunta você precisa primeiro estudar
os dois assuntos, não? Existem um monte de livros bons.

[]s, N.
=
Instruções para entrar na lista, sair da lista e usar a lista em
http://www.mat.puc-rio.br/~nicolau/olimp/obm-l.html
=r/~nicolau/olimp/obm-l.html
=











Yahoo! Messenger - Fale com seus amigos
online. Instale
agora!








[obm-l] Inversa e Transposta

2004-04-27 Por tôpico Cloves Jr



Mais uma de algera 
linear...

"Prove que, se A eh 
invertivel, entao A(t) eh invertivel e [A(t)] ^ -1 = (A 
^-1)(t)"

A(t) = transposta de 
A

[]s 


Cloves


[obm-l] Re: Relatividade

2004-04-27 Por tôpico Wallace Martins
Obrigado pelos esclarecimentos! 

Estive conversando com meu professor de Circuitos Elétricos (Prof. Mesquita) 
e ele estava me falando de coisas bem interessantes em relação à área de 
pesquisa sobre a velocidade da luz. Ele disse que ao assumirmos que a 
velocidade da luz vem variando ao longo do tempo e que, ainda hoje, pode ser 
suplantada chegamos a vários resultados bem mais consistentes com a 
prática. 

Wallace Martins
LPS/COPPE/UFRJ 

Paulo Santa Rita writes: 

Ola Wallace, 

Propor um problema e MUITO MAIS DIFICIL e MUITO MAIS IMPORTANTE que 
resolver um problema ... Isso pode soar paradoxal, pois, a priori, quem 
resolve um problema parece ter
mais habilidades e mais conhecimentos que aquele que tao somente o propos. 
Mas nas esferas
mais altas das ciencias, existem basicamente um tipo de problema : os 
ditos em aberto, em
torno dos quais gravitam boa parte das pesquisas de ponta. 

Um problema em aberto e basicamente um problema mal formulado... isto 
e, nos nao sabemos
ainda fazer as perguntas certas, implicando aparentes inconsistencias com 
o saber tradicional. Em
geral, quando a solucao surge, ela e fruto de uma colocacao original, 
fruto de uma pergunta idiota
no contexto da qual a verdadeira formulacao do problema aquire sentido e a 
sua solucao e bem
vislumbrada e bem encaminhada. 

Como exemplo podemos citar a equacao geral do 5 grau. Depois do sucesso em 
resolver a do
3 e do 4 grau, era natural que se procurasse a formula da equacao do 5 
grau. O problema
era basicamente esse : encontra uma expressao para as raizes em funcao dos 
coeficientes da
equacao. Hoje sabemos que esse problema so adquire uma resposta 
satisfatoria na Teoria dos
Grupos e que tal teoria jamais surge da postura ingenua ditada pelos 
sucessos com as equacoes
do 3 e do 4 grau. As perguntas precisam ser outras ... 

Fato semelhante ocorreu com as Geometrias nao-euclidianas. O problema 
era basicamente
esse : Prove que o 5 postulado de euclides e um teorema ou estabeleca em 
definitivo que
ele e um axioma. Hoje nos sabemos que esta linha de investigacao e 
anacronica, que os
desenvolvimentos frutiferos que dariam uma resposta satisfatoria a esta 
questao iriam surgir
de uma postura e formulacoes que nao guardavam relacoes diretas ou 
tangiveis com as
formulacoes originais. 

E poderiamos citar muitos outros exemplos mais modernos. A conviccao que 
fica e que, se
um problema esta em aberto e porque ele esta mal formulado, e porque 
estamos fazendo
as perguntas erradas e tal como Sisifo, enveredando por um caminho sem fim 
e sem
perspectivas. 

Em Matematica, a unica autoridade e a DEMONSTRACAO. Na Fisica, a 
EXPERIENCIA. Diante
dos fatos, nao pode haver argumento teorico contrario admissivel... Se um 
cientista faz um
experimento reprodutivel por qualquer outro cientista, estamos diante de 
um fato. Um fato !
Nao cabe-nos contesta-lo, cabe-nos compreende-lo. Se a nossa teoria, 
seja ela qual for,
for incompativel com ele, a  teoria esta errada, incompleta. O fato, nao : 
e um fato e ponto
final ! 

Entao, o que nos resta diante de um fato e tao somente aprender a fazer as 
perguntas
corretas ... Voce ja viu ou detectou um PRINCIPIO DA CAUSALIDADE ? Colocou 
ele dentro
de um tudo de ensaio e fez experiencias com ele ? Nao, certo ? Portanto, 
tudo que temos aqui
e uma fe, uma fe que, como toda fe ... pode ser malsa ... 

Note que o verdadeiro principio de causalidade que esta sendo derrogado e 
o PRINCIPIO DA
ANTECEDENCIA DAS CAUSAS, vale dizer, que toda causa deve anteceder, no 
tempo, os
seus efeitos. Mas o nexo causal se mantem, nao obstante estar invertido 
: a bomba explode
antes de nos apertarmos o detonador ! Mas ... Mas ... NOS PRECISAREMOS, 
AINDA, APERTAR
O DETONADOR ... 

Quero dizer que se voce ler nas entrelinhas, o que se chama QUEBRA DO 
PRINCIPIO DA
CAUSALIDADE  e apenas um suave e sutil PRINCIPIO DE NECESSIDADE ... Sim 
... Pois se
o nexo causal nao se mantiver, o paradoxa desaparece, isto e, so ha 
aparente absurdo
porque nos apertamos o detonador. Se a bomba explodisse e pouco depois 
ninguem
apertasse nenhum detonador, nada de anormal teria ocorrido : tao somente 
uma bomba
teria explodido e poderiamos remeter a ocorrencia a uma causa anterior 
qualquer e voltar
ao tradicional principio da causalidade : so ha paradoxo porque o nexo 
causal se mantem, vale
dizer, a bomba explode antes de eu apertar o detonador, mas ( e aqui ha um 
mais bem grande
que a maioria ainda nao viu ) eu NECESSITO apertar o detonador ... 

Na natureza existem muitas necessidades ... 

De maneira geral isso e claro no mundo organico, dos sistemas vivos. Um 
objeto qualquer,
identificavel e dotado de forma, fazendo parte de um contexto maior tambem 
identificavel,
cumpre um funcao ou executada movimentos somente plenamente 
compreenssiveis se
atentarmos para as funcoes maiores do contexto no qual ele se acha 
inserido, vale dizer,
alguams de suas propriedades sao irredutiveis ou incompreenssiveis se 
procurarmos principios ou 

Re: [obm-l] DUVIDA - funçao

2004-04-27 Por tôpico Artur Costa Steiner

--- Ricardo Bittencourt [EMAIL PROTECTED] wrote:
 João Silva wrote:
 
  - Uma função f : A -- B (em que A é o conjunto
 dos numeros reais 
  positivos não - nulos e B o conjunto dos reais) é
 estritamente crescente 
  e para x e y pertencentes a A temos: f (x.y) =
 f(x) + f(y) . Sabe-se 
  ainda que f(1) = 0 e f(2) = 1. Demonstrar que f(3)
 é irracional.
O fato de que f(1)=0 eh uma consequencia direta da
equacao funcional aa qual f satisfaz. Naum poderia ser
de outra forma. Se fizermos x=y =1, concluimos que
f(1*1) = f(1) = f(1) + f(1), = f(1) = 0. 

 
   f(sqrt(2)*sqrt(2))=2*f(sqrt(2))=f(2)=1
   logo f(sqrt(2))=1/2
 
   Daí fica claro que uma função f é o log na base 2
 né?
A rigor, para chegarmos a estah conclusao acho que
precisariamos assumir mais sobre f. Acho que soh a
equacao funcional dada naum basta. Se admitirmos
diferenciabilidade em um unico elemento de A, aih sim,
podemos afirmar que f e a funcao logaritmo na base 2. 

Assumindo-se que f eo logaritmo na base 2, a
irracionalidade de f(3) decorre da conclusao mais
geral de que, se m1 e n1 sao inteiros e log(n) m
(log de m na base n) naum for inteiro, entao log(n) m
eh irracional. Esta demonstracao jah foi apresentada
aqui na lista, acho que a mensagem tinha titulo
Logaritmo Irracional Acho que foi por volta de
setembro do ano passado.

Artur 




__
Do you Yahoo!?
Win a $20,000 Career Makeover at Yahoo! HotJobs  
http://hotjobs.sweepstakes.yahoo.com/careermakeover 
=
Instruções para entrar na lista, sair da lista e usar a lista em
http://www.mat.puc-rio.br/~nicolau/olimp/obm-l.html
=


[obm-l] Re: [obm-l] DUVIDA - funçao

2004-04-27 Por tôpico Cláudio \(Prática\)

- Original Message -
From: Ricardo Bittencourt [EMAIL PROTECTED]
To: [EMAIL PROTECTED]
Sent: Tuesday, April 27, 2004 1:52 PM
Subject: Re: [obm-l] DUVIDA - funçao


 João Silva wrote:

  - Uma função f : A -- B (em que A é o conjunto dos numeros reais
  positivos não - nulos e B o conjunto dos reais) é estritamente crescente
  e para x e y pertencentes a A temos: f (x.y) = f(x) + f(y) . Sabe-se
  ainda que f(1) = 0 e f(2) = 1. Demonstrar que f(3) é irracional.

 f(sqrt(2)*sqrt(2))=2*f(sqrt(2))=f(2)=1
 logo f(sqrt(2))=1/2

 Daí fica claro que uma função f é o log na base 2 né? Pois:

 log2 (1) = 0
 log2 (sqrt(2))=1/2
 log2 (2) = 1

 log2 (ab) = log2 (a) + log2 (b)

 Então resta provar que log2(3) é irracional.
 Pra isso acontecer, 3=2^(p/q) com p,q inteiros.

 Mas então 3^q=2^p, e com p,q inteiros isso é impossível.

 Hum.. resta provar que log2 é a única função f que
 satisfaz o enunciado, isso eu não sei fazer.

Oi, Ricardo:

Acho que o fato de f ser monótona crescente e satisfazer a f(xy) = f(x) +
f(y) implica que f é contínua.
Além disso, da mesma forma que na minha mensagem anterior, podemos provar
que se r é um racional qualquer, então:
f(2^r) = r*f(2) = r.
Finalmente, o conjunto dos números da forma 2^r (r racional) é denso em
(0,+infinito).
(dados a e b com 0  a  b, tome n = menor inteiro positivo tal que 2^(1/n)
 b/a e, uma vez fixado n, tome m = menor inteiro tal que 2^(m/n)  a.
Então, 2^((m-1)/n)  a  2^(m/n) = 2^((m-1)/n)*2^(1/n)  a*2^(1/n)  a*(b/a)
= b ).

Se g:(0,+infinito) - R é tal que:
g é estritamente crescente;
g(xy) = g(x) + g(y) para quaisquer x, y em (0,+infinito);
g(1) = 0 e g(2) = 1,
então, da mesma forma, g é contínua e g(2^r) = r para todo racional r.

Assim, a função F:(0,+infinito) - R dada por:
F(x) = f(x) - g(x)
é uma função contínua que se anula num subconjunto denso em (0,+infinito).
Logo, F é identicamente nula e, portanto, f é única.

[]s,
Claudio.


=
Instruções para entrar na lista, sair da lista e usar a lista em
http://www.mat.puc-rio.br/~nicolau/olimp/obm-l.html
=


[obm-l] So pra quem gosta de desafios

2004-04-27 Por tôpico Danilo notes

Construir uma função f de classe C^1 definida no intervalo [ 0 , infinito ) de modo que a função 
 a(t) = - f(t) g(t)/ { integral [ g(u) ] du } com u variando de 0 a t 

satisfaça as seguintes condições :


a(t) tende para o infinito quando t tende para o infinito 


a’(t) = C 0 para todo t suficientemente grande.

Onde g(t) = exponencial {integral [ f(s)] ds } com s variando de 0 a t.

 abs.Yahoo! Messenger - Fale com seus amigos online. Instale agora!

Re: [obm-l] Inversa e Transposta

2004-04-27 Por tôpico Alan Pellejero
cara, tem uma condição para um matriz ser inversível é que o determinante dela tem que ser  de 0...
outro teorema diz que o det. de uma matriz é igual ao determinante de sua inversa, entãi, a primeira parte da sua dúvida está respondida.
[
[A(t)] ^ -1 = (A ^-1)(t)' o inverso da tranposta = transposta do inverso...
isso você pode descobrir multiplicando ambos os membros pela transposta, inversa e ir trabalhando algebricamente...
Acho que é mais ou menos por ai
Abração
Alan Pellejero

Cloves Jr [EMAIL PROTECTED] wrote:


Mais uma de algera linear...

"Prove que, se A eh invertivel, entao A(t) eh invertivel e [A(t)] ^ -1 = (A ^-1)(t)"

A(t) = transposta de A

[]s 

ClovesYahoo! Messenger - Fale com seus amigos online. Instale agora!

[obm-l] Integral...

2004-04-27 Por tôpico Alan Pellejero
Olá amigos da lista, 
pessoal, gostaria de saber se alguém tem uma "carta na manga para esse aqui..."

/|1/ (senx + cosx) dx 
|
/

Eu fiz de uma maneira "corinthiana"...ou seja, deu 2 folhas!!!
Queria saber se alguém tem uma solução são-paulina (inteligente, rápida, objetiva, concisa...)

Ps: esse não é o da hipociclóide de novo.rs
valeu!
té mais!
Alan PellejeroYahoo! Messenger - Fale com seus amigos online. Instale agora!

Re: [obm-l] DUVIDA - Primo

2004-04-27 Por tôpico Thiago Ferraiol



Oi Dirichlet!
É... eu sei que não tem nada a ver com o problema 
inicial... só estou propondo um exercício que vi e não consegui resolver... 
tentei utilizar o pequeno teorema de Fermat e o teorema de Euler para numeros 
primos mas não obtive sucesso...

Será que pode me ajudar? Se possível, gostaria de 
uma sugestão para resolvê-lo... se mesmo assim não conseguir aí eu mando um 
recado novamente!!!

"Prove que se 2^n + 1 é primo, então n é potencia 
de 2!"

Obrigado!!!

  - Original Message - 
  From: 
  Johann Peter Gustav Lejeune 
  Dirichlet 
  To: [EMAIL PROTECTED] 
  Sent: Monday, April 26, 2004 1:11 
PM
  Subject: Re: [obm-l] DUVIDA - Primo
  
  Acho que isso nao tem nada a ver com o problema original...
  Se ce quer provar que existem infinitos primos, tem varios modos. O mais 
  legal e :prove elementarmente que a soma dos inversos dos primos diverge.Ou 
  reformulando em linguagem mais comum:
  
  Seja p(t) o t-esimo primo positivo.
  Seja S(t)=somatorio [1=x=t] (1/p(x)).
  Prove que para todo numero real M existe t natural (grande o bastante, 
  como ja era de se supor...)tal que S(t+x)M para todo x inteiro 
  nao-negativo.
  
  Va em frente e divirta-se!
  Ass.:JohannThiago Ferraiol 
  [EMAIL PROTECTED] wrote:
  
"uma outra interessante eh provar que se m e n sao inteiros 
positivosdistintos, entao os numeros 2^(2^m) + 1 e 2^(2^n) + 1 sao 
primos entre si)"...

Legal o problema... outro interessante é mostrar que se 2^n+1 é primo, 
então n é potencia de 2

Aguém tem alguma idéia!???Claudio Buffara 
[EMAIL PROTECTED] wrote:
on 
  24.04.04 22:55, [EMAIL PROTECTED] at 
  [EMAIL PROTECTED]wrote: Alguem pode me dar uma 
  ajuda nesta questão:  Seja p(n) o n-ésimo número 
  primo ( p(1) = 2, p(2) = 3, p(3) = 5 ...). Demonstrar que o 
  conjunto formado pelas diferenças p(n + 1) - p(n) possui um 
  numero infinito de elementos. [...]  Note que 
  isto equivale a provar que o conjunto das diferenças p(n+1)-p(n) 
  contém números arbitrariamente grandes, i.e. para todo N natural, 
  existem N naturais compostos consecutivos.  
  []s,Pegando um gancho na ideia do Fabio: um conjunto de inteiros 
  positivos ehinfinito se e somente se eh ilimitado. Assim, dado um 
  inteiro positivo nqualquer, precisamos mostrar que existem primos 
  consecutivos p e q tais queq - p  n.Por exemplo, 
  sejam:p = maior primo que eh menor do que (n+1)! + 2;q = menor 
  primo que eh maior do que (n+1)! + (n+1).Como (n+1)! + 2, (n+1)! + 3, 
  ..., (n+1)! + (n+1) sao n numeros compostosconsecutivos, temos que q - 
  p  n e que, se p  m  q, entao m eh composto.Pra essa 
  ideia funcionar, eh preciso que exista uma infinidade de primos,mas 
  isso pode ser provado independentemente (alem da 
  demonstracaoultra-tradicional de Euclides, a qual jah foi amplamente 
  discutida aqui nalista, uma outra interessante eh provar que se m e n 
  sao inteiros positivosdistintos, entao os numeros 2^(2^m) + 1 e 
  2^(2^n) + 1 sao primos entre 
  si)[]s,Claudio.=Instruções 
  para entrar na lista, sair da lista e usar a lista 
  emhttp://www.mat.puc-rio.br/~nicolau/olimp/obm-l.html= 
  
  
  Yahoo! 
  Messenger - Fale com seus amigos online. Instale 
  agora!


TRANSIRE SVVM PECTVS MVNDOQVE 
POTIRI
CONGREGATI EX TOTO ORBE MATHEMATICI OB 
SCRIPTA INSIGNIA TRIBVERE
Fields Medal(John Charles 
Fields)


Yahoo! 
Messenger - Fale com seus amigos online. Instale 
agora!


Re: [obm-l] Integral...

2004-04-27 Por tôpico niski
Voce nao tem que definir melhor o dominio disso ai?
e quando x é por exemplo 3pi/4?
Alan Pellejero wrote:
Olá amigos da lista,
pessoal, gostaria de saber se alguém tem uma carta na manga para esse 
aqui...
 
/
|1/ (senx + cosx) dx
|
/
--
Niski - http://www.linux.ime.usp.br/~niski
[upon losing the use of his right eye]
Now I will have less distraction
Leonhard Euler
=
Instruções para entrar na lista, sair da lista e usar a lista em
http://www.mat.puc-rio.br/~nicolau/olimp/obm-l.html
=


Re: [obm-l] DUVIDA - Primo

2004-04-27 Por tôpico Claudio Buffara
Title: Re: [obm-l] DUVIDA - Primo



on 27.04.04 17:23, Thiago Ferraiol at [EMAIL PROTECTED] wrote:

Oi Dirichlet!
É... eu sei que não tem nada a ver com o problema inicial... só estou propondo um exercício que vi e não consegui resolver... tentei utilizar o pequeno teorema de Fermat e o teorema de Euler para numeros primos mas não obtive sucesso...
 
Será que pode me ajudar? Se possível, gostaria de uma sugestão para resolvê-lo... se mesmo assim não conseguir aí eu mando um recado novamente!!!
 
Prove que se 2^n + 1 é primo, então n é potencia de 2!

 
Dica: suponha que n tem algum divisor impar maior do que 1 e veja o que acontece.

[]s,
Claudio.





Re: [obm-l] DUVIDA - Primo

2004-04-27 Por tôpico Domingos Jr.
2^{2k+1} = 2*4^k ~ 2(3+1)^k ~ 2 (mod 3)
logo 2^{2k+1} + 1 ~ 0 (mod 3) ou seja, se n é ímpar, 2^n + 1 é divisível por
3, então só para n = 1 temos 2^n+1 e 1 = 2^0.

suponha n = s*m, e s = 2^k, com k  0.

2^n + 1 = 2^(sm) + 1 = (2^s + 1)(2^{s(m-1)} - 2^{s(m-2)} + 2^{s(m-3)} -
... - 2^{s(1)} + 1)
portanto 2^n + 1 é composto se m  1

um exemplo da fatoração acima:
2^20 + 1 = 2^(5*2^2) + 1 = (2^4 + 1)(2^16 - 2^12 + 2^8 - 2^4 + 1)

[ ]'s

=
Instruções para entrar na lista, sair da lista e usar a lista em
http://www.mat.puc-rio.br/~nicolau/olimp/obm-l.html
=


Re: [obm-l] Integral...

2004-04-27 Por tôpico Claudio Buffara
Title: Re: [obm-l] Integral...



Aqui vai uma sugestao Botafoguense:

dx/(cos(x) + sen(x)) = 
(cos(x) - sen(x))*dx/(cos^2(x) - sen^2(x)) = 
(cos(x) - sen(x))*dx/(cos(2x)) =
cos(x)*dx/(1 - 2sen^2(x)) - sen(x)*dx/(2cos^2(x) - 1) (*)

Faca u = sen(x) e v = cos(x).
Entao, du = cos(x)*dx e dv = -sen(x)*dx.

Logo, (*) fica:
du/(1 - 2u^2) + dv/(2v^2 - 1) , o que talvez seja mais facil de tratar.

[]s,
Claudio.

on 27.04.04 17:36, Alan Pellejero at [EMAIL PROTECTED] wrote:


Olá amigos da lista, 
pessoal, gostaria de saber se alguém tem uma carta na manga para esse aqui...
 
/
| 1/ (senx + cosx) dx 
|
/
 
Eu fiz de uma maneira corinthiana...ou seja, deu 2 folhas!!!
Queria saber se alguém tem uma solução são-paulina (inteligente, rápida, objetiva, concisa...)
 
Ps: esse não é o da hipociclóide de novo.rs
valeu!
té mais!
Alan Pellejero


Yahoo! Messenger - Fale com seus amigos online. Instale agora! 






Re: [obm-l] Inversa e Transposta

2004-04-27 Por tôpico Johann Peter Gustav Lejeune Dirichlet
Isto sai direto da definiçao de produto de matrizes!Cloves Jr [EMAIL PROTECTED] wrote:


Mais uma de algera linear...

"Prove que, se A eh invertivel, entao A(t) eh invertivel e [A(t)] ^ -1 = (A ^-1)(t)"

A(t) = transposta de A

[]s 

Cloves

TRANSIRE SVVM PECTVS MVNDOQVE POTIRI
CONGREGATI EX TOTO ORBE MATHEMATICI OB SCRIPTA INSIGNIA TRIBVERE
Fields Medal(John Charles Fields)Yahoo! Messenger - Fale com seus amigos online. Instale agora!

Re: [obm-l] DUVIDA - Primo

2004-04-27 Por tôpico Johann Peter Gustav Lejeune Dirichlet
E por isso que eu ainda recomendo uma Eureka!
LEMA:a+1 divide a^(impar)+1.

Prove-o!
Agora suponha que n=ik com i impar e escreva (2^k)^i+1.Agora foi!Thiago Ferraiol [EMAIL PROTECTED] wrote:




Oi Dirichlet!
É... eu sei que não tem nada a ver com o problema inicial... só estou propondo um exercício que vi e não consegui resolver... tentei utilizar o pequeno teorema de Fermat e o teorema de Euler para numeros primos mas não obtive sucesso...

Será que pode me ajudar? Se possível, gostaria de uma sugestão para resolvê-lo... se mesmo assim não conseguir aí eu mando um recado novamente!!!

"Prove que se 2^n + 1 é primo, então n é potencia de 2!"

Obrigado!!!

- Original Message - 
From: Johann Peter Gustav Lejeune Dirichlet 
To: [EMAIL PROTECTED] 
Sent: Monday, April 26, 2004 1:11 PM
Subject: Re: [obm-l] DUVIDA - Primo

Acho que isso nao tem nada a ver com o problema original...
Se ce quer provar que existem infinitos primos, tem varios modos. O mais legal e :prove elementarmente que a soma dos inversos dos primos diverge.Ou reformulando em linguagem mais comum:

Seja p(t) o t-esimo primo positivo.
Seja S(t)=somatorio [1=x=t] (1/p(x)).
Prove que para todo numero real M existe t natural (grande o bastante, como ja era de se supor...)tal que S(t+x)M para todo x inteiro nao-negativo.

Va em frente e divirta-se!
Ass.:JohannThiago Ferraiol [EMAIL PROTECTED] wrote:

"uma outra interessante eh provar que se m e n sao inteiros positivosdistintos, entao os numeros 2^(2^m) + 1 e 2^(2^n) + 1 sao primos entre si)"...

Legal o problema... outro interessante é mostrar que se 2^n+1 é primo, então n é potencia de 2

Aguém tem alguma idéia!???Claudio Buffara [EMAIL PROTECTED] wrote:
on 24.04.04 22:55, [EMAIL PROTECTED] at [EMAIL PROTECTED]wrote: Alguem pode me dar uma ajuda nesta questão:  Seja p(n) o n-ésimo número primo ( p(1) = 2, p(2) = 3, p(3) = 5 ...). Demonstrar que o conjunto formado pelas diferenças p(n + 1) - p(n) possui um numero infinito de elementos. [...]  Note que isto equivale a provar que o conjunto das diferenças p(n+1)-p(n) contém números arbitrariamente grandes, i.e. para todo N natural, existem N naturais compostos consecutivos.  []s,Pegando um gancho na ideia do Fabio: um conjunto de inteiros positivos ehinfinito se e somente se eh ilimitado. Assim, dado um inteiro positivo nqualquer, precisamos mostrar que existem primos consecutivos p e q tais queq - p  n.Por
 exemplo, sejam:p = maior primo que eh menor do que (n+1)! + 2;q = menor primo que eh maior do que (n+1)! + (n+1).Como (n+1)! + 2, (n+1)! + 3, ..., (n+1)! + (n+1) sao n numeros compostosconsecutivos, temos que q - p  n e que, se p  m  q, entao m eh composto.Pra essa ideia funcionar, eh preciso que exista uma infinidade de primos,mas isso pode ser provado independentemente (alem da demonstracaoultra-tradicional de Euclides, a qual jah foi amplamente discutida aqui nalista, uma outra interessante eh provar que se m e n sao inteiros positivosdistintos, entao os numeros 2^(2^m) + 1 e 2^(2^n) + 1 sao primos entre si)[]s,Claudio.=Instruções para entrar na lista, sair da lista e usar a lista emhttp://www.mat.puc-rio.br/~nicolau/olimp/obm-l.html= 


Yahoo! Messenger - Fale com seus amigos online. Instale agora!


TRANSIRE SVVM PECTVS MVNDOQVE POTIRI
CONGREGATI EX TOTO ORBE MATHEMATICI OB SCRIPTA INSIGNIA TRIBVERE
Fields Medal(John Charles Fields)


Yahoo! Messenger - Fale com seus amigos online. Instale agora!

TRANSIRE SVVM PECTVS MVNDOQVE POTIRI
CONGREGATI EX TOTO ORBE MATHEMATICI OB SCRIPTA INSIGNIA TRIBVERE
Fields Medal(John Charles Fields)Yahoo! Messenger - Fale com seus amigos online. Instale agora!

[obm-l] Geometria Plana - Desafio (?)

2004-04-27 Por tôpico Victor Machado



Bom, esta questão foi um desafio para mim, não sei 
para os senhores :

Dado um trapézio ABCD de bases AB= a e CD=b e os 
pontos M e N pertencentes aos lados NÃO-paralelos. Se o segmento MN divide esse 
trapézio em dois outros trapézios equivalentes, calcule MN em função dos lados 
AB=a e CD=b.

Victor.


Re: [obm-l] Integral...

2004-04-27 Por tôpico Johann Peter Gustav Lejeune Dirichlet
Essa e facil!
Faça t= tg (x/2).Tente escrevr sen x, cos x e dx como funçoes de t (isso e facil mesmo.Tente procurar em um livro de analise!)
Ai e so decompor em fraçoes.
PS.:Com este seu comentario voce estaria insultando Erdös.Alan Pellejero [EMAIL PROTECTED] wrote:

Olá amigos da lista, 
pessoal, gostaria de saber se alguém tem uma "carta na manga para esse aqui..."

/|1/ (senx + cosx) dx 
|
/

Eu fiz de uma maneira "corinthiana"...ou seja, deu 2 folhas!!!
Queria saber se alguém tem uma solução são-paulina (inteligente, rápida, objetiva, concisa...)

Ps: esse não é o da hipociclóide de novo.rs
valeu!
té mais!
Alan Pellejero


Yahoo! Messenger - Fale com seus amigos online. Instale agora!

TRANSIRE SVVM PECTVS MVNDOQVE POTIRI
CONGREGATI EX TOTO ORBE MATHEMATICI OB SCRIPTA INSIGNIA TRIBVERE
Fields Medal(John Charles Fields)Yahoo! Messenger - Fale com seus amigos online. Instale agora!

Re: [obm-l] Integral...

2004-04-27 Por tôpico Johann Peter Gustav Lejeune Dirichlet
Meu, ce ainda se apega a detalhes???niski [EMAIL PROTECTED] wrote:
Voce nao tem que definir melhor o dominio disso ai?e quando x é por exemplo 3pi/4?Alan Pellejero wrote:  Olá amigos da lista, pessoal, gostaria de saber se alguém tem uma "carta na manga para esse  aqui..."  / | 1/ (senx + cosx) dx | /-- Niski - http://www.linux.ime.usp.br/~niski[upon losing the use of his right eye]"Now I will have less distrraction"Leonhard Euler=Instruções para entrar na lista, sair da lista e usar a lista emhttp://www.mat.puc-rio.br/~nicolau/olimp/obm-l.html=

TRANSIRE SVVM PECTVS MVNDOQVE POTIRI
CONGREGATI EX TOTO ORBE MATHEMATICI OB SCRIPTA INSIGNIA TRIBVERE
Fields Medal(John Charles Fields)Yahoo! Messenger - Fale com seus amigos online. Instale agora!

Re: [obm-l] Integral...

2004-04-27 Por tôpico Marcio Cohen



 Sim. sqrt(2)senx + sqrt(2)cosx = 
2sen(x+45). Isso ajuda bastante se voce ja sabe a integral de secante de cabeça 
(será q existe alguem nesse mundoque nunca reparou que a derivada de 
ln(sec+tg) eh (sec*tg + sec^2)/(sec+tg) = sec ?).

  - Original Message - 
  From: 
  Alan Pellejero 
  To: [EMAIL PROTECTED] 
  Sent: Tuesday, April 27, 2004 5:36 
  PM
  Subject: [obm-l] Integral...
  
  Olá amigos da lista, 
  pessoal, gostaria de saber se alguém tem uma "carta na manga para esse 
  aqui..."
  
  /|1/ (senx + cosx) dx 
  |
  /
  
  Eu fiz de uma maneira "corinthiana"...ou seja, deu 2 folhas!!!
  Queria saber se alguém tem uma solução são-paulina (inteligente, rápida, 
  objetiva, concisa...)
  
  Ps: esse não é o da hipociclóide de novo.rs
  valeu!
  té mais!
  Alan Pellejero
  
  
  Yahoo! 
  Messenger - Fale com seus amigos online. Instale 
  agora!


Re: [obm-l] Integral...

2004-04-27 Por tôpico Alan Pellejero
desculpe-me, mas meu professor de cálculo passou assim...
niski [EMAIL PROTECTED] wrote:
Voce nao tem que definir melhor o dominio disso ai?e quando x é por exemplo 3pi/4?Alan Pellejero wrote:  Olá amigos da lista, pessoal, gostaria de saber se alguém tem uma "carta na manga para esse  aqui..."  / | 1/ (senx + cosx) dx | /-- Niski - http://www.linux.ime.usp.br/~niski[upon losing the use of his right eye]"Now I will have less distrraction"Leonhard Euler=Instruções para entrar na lista, sair da lista e usar a lista emhttp://www.mat.puc-rio.br/~nicolau/olimp/obm-l.html=Yahoo!
 Messenger - Fale com seus amigos online. Instale agora!

[obm-l] Questao da Eureka 01

2004-04-27 Por tôpico Faelccmm
Ola pessoal,

Poderiam me explicar como se resolve esta:

1) A equacao do 2 grau ax^2 + bx  3 = 0 tem 1 como uma de suasraizes. Sabendo que os coeficientes a e b sao numeros primos positivos, podemos afirmar que a^2 + b^2 eh igual a: a) 29 b) 89 c) 17 d) 13 e) 53 Ps: Alguem poderia me enviar a figura a que se refere a questao de treinamento (numero 02) da eureka 01 ? Fiz o download da revista, mas nao aparece esta figura.


Re: [obm-l] Integral...

2004-04-27 Por tôpico niski
Pra ser sincero eu percebi num relance que a resposta é
(1/sqrt(2))*(ln(2-sqrt(2)+sqrt(2)*tg(x/2))-ln(2+sqrt(2)-sqrt(2)*tg(x/2)))
mas como voces, seres humanos, ainda nao estao preparados para saber 
como eu fiz isso, resolvi desviar do assunto principal e me apegar aos 
detalhes.

Johann Peter Gustav Lejeune Dirichlet wrote:
Meu, ce ainda se apega a detalhes???
niski [EMAIL PROTECTED] wrote:
Voce nao tem que definir melhor o dominio disso ai?
e quando x é por exemplo 3pi/4?
--
Niski - http://www.linux.ime.usp.br/~niski
[upon losing the use of his right eye]
Now I will have less distraction
Leonhard Euler
=
Instruções para entrar na lista, sair da lista e usar a lista em
http://www.mat.puc-rio.br/~nicolau/olimp/obm-l.html
=


Re: [obm-l] Integral...

2004-04-27 Por tôpico niski
Mas como voce resolveu se eu fiz uma observacao sem o menor sentido e 
voce (parece) que nem reparou?

Alan Pellejero wrote:
desculpe-me, mas meu professor de cálculo passou assim...

--
Niski - http://www.linux.ime.usp.br/~niski
[upon losing the use of his right eye]
Now I will have less distraction
Leonhard Euler
=
Instruções para entrar na lista, sair da lista e usar a lista em
http://www.mat.puc-rio.br/~nicolau/olimp/obm-l.html
=


Re: [obm-l] Inversa e Transposta + FUNCAO EUREKA

2004-04-27 Por tôpico Villard
Para ficar mais fácil de escrever, seja B = A^(-1). Quero
mostrar que B^t=(A^t)^(-1), ou seja, que B^t * A^t = IMas isso é
verdade, pois B^t * A^t = (A*B)^t = I^t = I , pois B é a inversa de A.
Bem, pessoal, eu andei vendo alguns discutindo o problema 83 da
eureka, aquele das funções : f(2003) = 2003, f(m)=2003 para todo m =
2003, f(m+f(n))=f(f(m)) + f(n).Eu achei 3 funções...Eram f(m) =
m, f(m) = 2003*(1+parte inteira [(m-1)/2003] ),f(m) = 2003*(parte
inteira [m/2003]).Vou ver se escrevo a minha solução num email ainda
hj.Abraços, Villard

- Mensagem Original De:
[EMAIL PROTECTED]Para: "Grupo OBM"
[EMAIL PROTECTED]Assunto: [obm-l] Inversa e
TranspostaData: 27/04/04 16:00

Mais uma de
algera linear...

"Prove que, se A
eh invertivel, entao A(t) eh invertivel e [A(t)] ^ -1 = (A
^-1)(t)"

A(t) =
transposta de A

[]s


Cloves

=
Instruções para entrar na lista, sair da lista e usar a lista em
http://www.mat.puc-rio.br/~nicolau/olimp/obm-l.html
=


[obm-l] Problema da função da eureka 18

2004-04-27 Por tôpico Villard
Como prometido, segue minha solução para o problema 83 da
eureka 18 (colocarei resumida, pois é meio longa):Seja (*)
f(m+f(n))=f(f(m)) + f(n). Faça m=n=0: Isso nos dá f(0)=0.Faça n=0:
f(m)=f(f(m)). Então (*) vira f(m+f(n)) = f(m)+f(n).Seja I =
{0,1,2,...,2002}.Caso 1: f se anula todo o conjunto I.Fazendo
n=2003, temos que f(m+2003) = f(m) + 2003, logo os valores de f são 0,0,
...,0,2003,2003, ...,2003,4006,4006, ... 4006,6009, ou seja, f(m) =
2003*(parte inteira [m/2003])Caso 2: f não se anula em todo o
conjunto I.Seja u o menor valor positivode f(I). Então existe k em
I tal que f(k) = u. Fazendo n = k, temos que f(m+u)=f(m)+u . Usando isso
repetidas vezes, temos que f(j*u) = j*u, para todo inteiro j. Agora
divida 2003 por u: Temos que 2003 = q *u+r, onde 0=ru. Então,
fazendo m=r e n=qu, temos que f(r+f(qu)) = f(r)+f(qu), logo
f(r+qu) = f(r) + qu, logo f(2003) = f(r) + qu, ou seja, f(r) = 2003 - qu =
r, logo f(r) = r. Como r está na imagem de f, temos que r = 0, ou r
= u (este segundo caso não pode, pois r  u). Logo, u é divisor de
2003, portanto (como 2003 é primo), temos que u=1 ou u=2003.se u = 2003
(como é mínimo), temos que os valores de f são 0, 2003, 2003, ...,2003,
4006,4006, ..., 4006, 6009,... ou seja, f(m) = 2003*(1+parte
inteira[(m-1)/2003]).se u = 1, temos que f(m+1) = f(m) + 1, logo (por
indução), temos que f(m) = m.Isso dá o total de 3
funções.Abraços, Villard

- Mensagem Original De:
[EMAIL PROTECTED]Para: "[EMAIL PROTECTED]"
[EMAIL PROTECTED]Assunto: [obm-l] Eureka 18 e Olimpiada
CearenseData: 25/04/04 23:18 então parece que qualquer valor de k serve, mas f(1) =
2003, então temos 2004 valores para f(1), cada um determinando uma
função diferente. acho que é isso...opa, mas f(2003)
= 20032003 = q*k + r = f(2003) = f(q*k + r) = (q + r)k = r =
0 = k|2003então temos que tomar f(1) como divisor de
2003desculpem pelo erro bobo, espero que agora sim esteja
correto!=Instruções
para entrar na lista, sair da lista e usar a lista emhttp://www.mat.puc-rio.br/~nicolau/olimp/obm-l.html=

=
Instruções para entrar na lista, sair da lista e usar a lista em
http://www.mat.puc-rio.br/~nicolau/olimp/obm-l.html
=


Re: [obm-l] Re: [obm-l] DUVIDA - funçao

2004-04-27 Por tôpico Claudio Buffara
on 27.04.04 15:25, Cláudio (Prática) at [EMAIL PROTECTED]
wrote:

 
 - Original Message -
 From: Ricardo Bittencourt [EMAIL PROTECTED]
 To: [EMAIL PROTECTED]
 Sent: Tuesday, April 27, 2004 1:52 PM
 Subject: Re: [obm-l] DUVIDA - funçao
 
 
 João Silva wrote:
 
 - Uma função f : A -- B (em que A é o conjunto dos numeros reais
 positivos não - nulos e B o conjunto dos reais) é estritamente crescente
 e para x e y pertencentes a A temos: f (x.y) = f(x) + f(y) . Sabe-se
 ainda que f(1) = 0 e f(2) = 1. Demonstrar que f(3) é irracional.
 
 f(sqrt(2)*sqrt(2))=2*f(sqrt(2))=f(2)=1
 logo f(sqrt(2))=1/2
 
 Daí fica claro que uma função f é o log na base 2 né? Pois:
 
 log2 (1) = 0
 log2 (sqrt(2))=1/2
 log2 (2) = 1
 
 log2 (ab) = log2 (a) + log2 (b)
 
 Então resta provar que log2(3) é irracional.
 Pra isso acontecer, 3=2^(p/q) com p,q inteiros.
 
 Mas então 3^q=2^p, e com p,q inteiros isso é impossível.
 
 Hum.. resta provar que log2 é a única função f que
 satisfaz o enunciado, isso eu não sei fazer.
 
 Oi, Ricardo:
 
 Acho que o fato de f ser monótona crescente e satisfazer a f(xy) = f(x) +
 f(y) implica que f é contínua.
 Além disso, da mesma forma que na minha mensagem anterior, podemos provar
 que se r é um racional qualquer, então:
 f(2^r) = r*f(2) = r.
 Finalmente, o conjunto dos números da forma 2^r (r racional) é denso em
 (0,+infinito).
 (dados a e b com 0  a  b, tome n = menor inteiro positivo tal que 2^(1/n)
  b/a e, uma vez fixado n, tome m = menor inteiro tal que 2^(m/n)  a.
 Então, 2^((m-1)/n)  a  2^(m/n) = 2^((m-1)/n)*2^(1/n)  a*2^(1/n)  a*(b/a)
 = b ).
 
 Se g:(0,+infinito) - R é tal que:
 g é estritamente crescente;
 g(xy) = g(x) + g(y) para quaisquer x, y em (0,+infinito);
 g(1) = 0 e g(2) = 1,
 então, da mesma forma, g é contínua e g(2^r) = r para todo racional r.
 
 Assim, a função F:(0,+infinito) - R dada por:
 F(x) = f(x) - g(x)
 é uma função contínua que se anula num subconjunto denso em (0,+infinito).
 Logo, F é identicamente nula e, portanto, f é única.
 
 
Consegui provar que f eh continua, o que completa a demonstracao de que f eh
unica (e, portanto, igual a funcao logaritmo de base 2).

A demonstracao baseia-se nos seguintes fatos:
f(4^(1/n)) = 2/n  e  f(1/4)^(1/n)) = -2/n
e
n = 2  ==  (1/4)^(1/n) = 1 - 1/n  e  1 + 1/n  4^(1/n)

Seja a um real positivo.

Dado eps  0, tomemos um inteiro positivo n = 2 tal que:
1/n  eps/2 == 2/n  eps.

Entao:
|x - a|  a/n ==

|x/a - 1|  1/n ==

1 - 1/n  x/a  1 + 1/n ==

(1/4)^(1/n)  x/a  4^(1/n) ==

f((1/4)^(1/n))  f(x/a)  f(4^(1/n)) ==

-2/n  f(x/a)  2/n

|f(x/a)|  2/n ==

|f(x) - f(a)|  2/n  eps


[]s,
Claudio.


=
Instruções para entrar na lista, sair da lista e usar a lista em
http://www.mat.puc-rio.br/~nicolau/olimp/obm-l.html
=


Re: [obm-l] COMBINATÓRIA e engraçadinhos

2004-04-27 Por tôpico Augusto Cesar de Oliveira Morgado
Eu NAO mandei a mensagem abaixo.
Morgado

==
Mensagem  enviada  pelo  CIP  WebMAIL  - Nova Geração - v. 2.1
CentroIn Internet Provider  http://www.centroin.com.br
Tel: (21) 2542-4849, (21) 2295-3331Fax: (21) 2295-2978
Empresa 100% Brasileira - Desde 1992 prestando servicos online


-- Original Message ---
From: Augusto Cesar de Oliveira Morgado [EMAIL PROTECTED]
To: [EMAIL PROTECTED]
Sent: Tue, 27 Apr 2004 09:07:19 -0200
Subject: Re: [obm-l] COMBINATÓRIA

 Claúdio, 
 O Nicolau resolveu e discutiu a questão proposta na olimpíada 
 cearense. Está resolvida no link abaixo, está resolvida em três etapas.
 http://www.mat.puc-rio.br/~nicolau/olimp/obm-l.200401/msg00368.html
 
 --=_NextPart_000_0025_01C42B28.53AAB880
 Content-Type: text/html;
   charset=iso-8859-1
 Content-Transfer-Encoding: quoted-printable
 
 !DOCTYPE HTML PUBLIC -//W3C//DTD HTML 4.0 Transitional//EN
 HTMLHEAD
 META http-equiv=Content-Type content=text/html; charset=iso-8859-
 1 META content=MSHTML 5.50.4134.100 name=GENERATOR STYLE/STYLE
 /HEAD
 BODY bgColor=#ff
 DIVFONT face=Arial size=2FONT face=Times New Roman size=3/FONT
 DIVClaúdio, /DIV
 DIVOnbsp;Nicolau resolveu e discutiu anbsp;questão proposta na 
 olimpíada cearense. Está resolvida no link abaixo, está resolvida em 
 três etapas./DIV DIVA href=http://www.mat.puc-
 rio.br/~nicolau/olimp/obm-l.200401/msg00368.htmlhttp://www.mat.puc-
 rio.br/~nicolau/olimp/obm-
l.200401/msg00368.html/A/DIV/FONT/DIV/BODY/HTML
 
 --=_NextPart_000_0025_01C42B28.53AAB880--
 
Instru
gues para entrar na lista, sair da lista e usar a lista em
 http://www.mat.puc-rio.br/~nicolau/olimp/obm-l.html
 
--- End of Original Message ---

=
Instruções para entrar na lista, sair da lista e usar a lista em
http://www.mat.puc-rio.br/~nicolau/olimp/obm-l.html
=


Re: [obm-l] Integral...

2004-04-27 Por tôpico Claudio Buffara
Title: Re: [obm-l] Integral...



on 27.04.04 18:42, Marcio Cohen at [EMAIL PROTECTED] wrote

Será q existe alguem nesse mundo que nunca reparou que a derivada de ln(sec+tg) eh:
 (sec*tg + sec^2)/(sec+tg) = sec ?).

Sendo muito otimista, eu diria que pelo menos 5 bilhoes de pessoas nao sabem nem o que eh secante, muito menos que ela eh a derivada de ln(sec + tg).

[]s,
Claudio.






Re: [obm-l] Questao da Eureka 01

2004-04-27 Por tôpico Claudio Buffara
Title: Re: [obm-l] Questao da Eureka 01



on 27.04.04 19:04, [EMAIL PROTECTED] at [EMAIL PROTECTED] wrote:

Ola pessoal, 

Poderiam me explicar como se resolve esta: 

1) A equacao do 2º grau ax^2 + bx ­ 3 = 0 tem ­1 como uma de suas
raizes. Sabendo que os coeficientes a e b sao numeros primos positivos, podemos afirmar que a^2 + b^2 eh igual a:
 
a) 29 b) 89 c) 17 d) 13 e) 53
 
as raizes sao -1 a u == 
u - 1 = -b/a e -u = -3/a ==
3/a - 1 = -b/a ==
3 - a = -b ==
a = b + 3 ==
a  b e a, b tem paridades distintas.

Como eles sao primos positivos, soh pode ser b = 2 e a = 5 ==
a^2 + b^2 = 25 + 4 = 29

[]s,
Claudio.





Re: [obm-l] COMBINATÓRIA e engraçadinhos

2004-04-27 Por tôpico Claudio Buffara
Muito esquisito! Eu recebi essa mensagem mas o remetente foi o Max
[EMAIL PROTECTED], que perguntou recentemente sobre o problema, e nao
o Morgado.


on 27.04.04 22:35, Augusto Cesar de Oliveira Morgado at
[EMAIL PROTECTED] wrote:

 Eu NAO mandei a mensagem abaixo.
 Morgado
 
 ==
 Mensagem  enviada  pelo  CIP  WebMAIL  - Nova Geração - v. 2.1
 CentroIn Internet Provider  http://www.centroin.com.br
 Tel: (21) 2542-4849, (21) 2295-3331Fax: (21) 2295-2978
 Empresa 100% Brasileira - Desde 1992 prestando servicos online
 
 
 -- Original Message ---
 From: Augusto Cesar de Oliveira Morgado [EMAIL PROTECTED]
 To: [EMAIL PROTECTED]
 Sent: Tue, 27 Apr 2004 09:07:19 -0200
 Subject: Re: [obm-l] COMBINATÓRIA
 
 Claúdio, 
 O Nicolau resolveu e discutiu a questão proposta na olimpíada
 cearense. Está resolvida no link abaixo, está resolvida em três etapas.
 http://www.mat.puc-rio.br/~nicolau/olimp/obm-l.200401/msg00368.html
 
 --=_NextPart_000_0025_01C42B28.53AAB880
 Content-Type: text/html;
 charset=iso-8859-1
 Content-Transfer-Encoding: quoted-printable
 
 !DOCTYPE HTML PUBLIC -//W3C//DTD HTML 4.0 Transitional//EN
 HTMLHEAD
 META http-equiv=Content-Type content=text/html; charset=iso-8859-
 1 META content=MSHTML 5.50.4134.100 name=GENERATOR STYLE/STYLE
 /HEAD
 BODY bgColor=#ff
 DIVFONT face=Arial size=2FONT face=Times New Roman size=3/FONT
 DIVClaúdio, /DIV
 DIVOnbsp;Nicolau resolveu e discutiu anbsp;questão proposta na
 olimpíada cearense. Está resolvida no link abaixo, está resolvida em
 três etapas./DIV DIVA href=http://www.mat.puc-
 rio.br/~nicolau/olimp/obm-l.200401/msg00368.htmlhttp://www.mat.puc-
 rio.br/~nicolau/olimp/obm-
 l.200401/msg00368.html/A/DIV/FONT/DIV/BODY/HTML
 
 --=_NextPart_000_0025_01C42B28.53AAB880--
 
 Instru
 gues para entrar na lista, sair da lista e usar a lista em
 http://www.mat.puc-rio.br/~nicolau/olimp/obm-l.html
 
 --- End of Original Message ---
 
 =
 Instruções para entrar na lista, sair da lista e usar a lista em
 http://www.mat.puc-rio.br/~nicolau/olimp/obm-l.html
 =
 


=
Instruções para entrar na lista, sair da lista e usar a lista em
http://www.mat.puc-rio.br/~nicolau/olimp/obm-l.html
=


[obm-l] Binomios... Duvida ( interessante )

2004-04-27 Por tôpico Fabio Contreiras



1) Determinar o coeficiente de x^3 no 
desenvolvimento de ( 2x - 3 )^4 . ( x + 2 )^5


Alguem pode me explicar o caminho ?
Abração!


Re: [obm-l] Inversa e Transposta

2004-04-27 Por tôpico Marco Silva
 --- Cloves Jr [EMAIL PROTECTED] escreveu:  Mais uma
de algera linear...
 
 Prove que, se A eh invertivel, entao A(t) eh
 invertivel e [A(t)] ^ -1 = (A
 ^ -1)(t)
 
 A(t) = transposta de A
 
 []s
 
 Cloves
 ---
 Outgoing mail is certified Virus Free.
 Checked by AVG anti-virus system
 (http://www.grisoft.com).
 Version: 6.0.669 / Virus Database: 431 - Release
 Date: 26/04/04


  Se existe A inversa de A entao

(A*A^-1)(t) = A^-1(t)*A(t) = ( A^-1(t) ) * A(t) = I
(a transposta de I eh I), observando a ultima equação
temos que A^-1(t) eh a inversa de A(t), isto eh

A^-1(t)= A(t)^-1

A ultima equacao tb prova que A(t) eh invertivel.

[]s

Marco Arthur

__

Yahoo! Mail - O melhor e-mail do Brasil! Abra sua conta agora:
http://br.yahoo.com/info/mail.html
=
Instruções para entrar na lista, sair da lista e usar a lista em
http://www.mat.puc-rio.br/~nicolau/olimp/obm-l.html
=


Re:[obm-l] Geometria Plana - Desafio (?)

2004-04-27 Por tôpico rickufrj
-- Início da mensagem original ---

  De: [EMAIL PROTECTED]
Para: [EMAIL PROTECTED]
  Cc: 
Data: Tue, 27 Apr 2004 18:42:49 -0300
 Assunto: [obm-l] Geometria Plana - Desafio (?)

 Bom, esta questão foi um desafio para mim, não sei 
para os senhores :
 
 Dado um trapézio ABCD de bases AB= a e CD=b e os 
pontos M e N pertencentes aos lados NÃO-paralelos. Se 
o segmento MN divide esse trapézio em dois outros 
trapézios equivalentes, calcule MN em função dos lados 
AB=a e CD=b.
 
 Victor.
 


=

Olá Victor, esse acho que é do Naval de 99.
Na verdade, ai vai uma dica:
Em uma prova de múltipla escolha, como a do Naval, 
quando o enunciado for parecido com este, ou seja, 
pede a prova de uma relação na maioria dos casos 
geométrica , para valer para todos os casos,ela deve 
ser genérica , você pode provar para um caso 
particular , que fatalmente valerá para o caso 
genérico . Isso vem da seguinte suposição:
Em uma prova de múltipla escolha, normalmente quando 
se pede para encontrar uma relação é porque ela é 
verdadeira para todos os casos, incluindo assim o caso 
particular que será mais fácil de fazer. 
Mas o problema em questão acho que não muda muita 
coisa, pois é um pouco trivial-não utiliza nenhuma 
idéia esperta- mas na prova do CN de 01, se não me 
engano , tem um da área do triângulo que ajuda muito 
considerar o triângulo eqüilátero e se for tentar 
fazer com um triângulo qualquer , tem umas idéias 
maneirinhas.
Abraços
Luiz H. Barbosa

 
__
Acabe com aquelas janelinhas que pulam na sua tela.
AntiPop-up UOL - É grátis!
http://antipopup.uol.com.br/



=
Instruções para entrar na lista, sair da lista e usar a lista em
http://www.mat.puc-rio.br/~nicolau/olimp/obm-l.html
=


Re:[obm-l] Binomios... Duvida ( interessante )

2004-04-27 Por tôpico rickufrj
-- Início da mensagem original ---

  De: [EMAIL PROTECTED]
Para: [EMAIL PROTECTED]
  Cc: 
Data: Tue, 27 Apr 2004 22:44:45 -0300
 Assunto: [obm-l] Binomios... Duvida ( interessante )

 1) Determinar o coeficiente de x^3 no 
desenvolvimento de ( 2x - 3 )^4 . ( x + 2 )^5
 
 
 Alguem pode me explicar o caminho ?
 Abração!


Olhe para :
x^3=(x^0)*(x^3)=(x^1)*(x^2)=(x^2)*(x^1)=(x^3)*(x^0)
Com isso vc calcula os coeficientes de cada um e soma !
Abraços
Luiz H. Barbosa
 
__
Acabe com aquelas janelinhas que pulam na sua tela.
AntiPop-up UOL - É grátis!
http://antipopup.uol.com.br/



=
Instruções para entrar na lista, sair da lista e usar a lista em
http://www.mat.puc-rio.br/~nicolau/olimp/obm-l.html
=


Re: Re:[obm-l] Binomios... Duvida ( interessante )

2004-04-27 Por tôpico Fellipe Rossi
Só pra esclarescer um pouco mais
O que o rick quis dizer eh q qdo multiplicamos o x^0 com o x^3 obtemos um
coeficiente de grau 3, o mesmo para 1 e 2 e pra todos q ele exemplificou.
Assim, seu trabalho sera achar cada coeficiente em cada polinomio e depois
multiplicar os q equivalerão a 3...
meio braçal isto hehe


- Original Message -
From: rickufrj [EMAIL PROTECTED]
To: obm-l [EMAIL PROTECTED]
Sent: Tuesday, April 27, 2004 11:17 PM
Subject: Re:[obm-l] Binomios... Duvida ( interessante )


 -- Início da mensagem original ---

   De: [EMAIL PROTECTED]
 Para: [EMAIL PROTECTED]
   Cc:
 Data: Tue, 27 Apr 2004 22:44:45 -0300
  Assunto: [obm-l] Binomios... Duvida ( interessante )

  1) Determinar o coeficiente de x^3 no
 desenvolvimento de ( 2x - 3 )^4 . ( x + 2 )^5
 
 
  Alguem pode me explicar o caminho ?
  Abração!

 
 Olhe para :
 x^3=(x^0)*(x^3)=(x^1)*(x^2)=(x^2)*(x^1)=(x^3)*(x^0)
 Com isso vc calcula os coeficientes de cada um e soma !
 Abraços
 Luiz H. Barbosa

 __
 Acabe com aquelas janelinhas que pulam na sua tela.
 AntiPop-up UOL - É grátis!
 http://antipopup.uol.com.br/



 =
 Instruções para entrar na lista, sair da lista e usar a lista em
 http://www.mat.puc-rio.br/~nicolau/olimp/obm-l.html
 =




=
Instruções para entrar na lista, sair da lista e usar a lista em
http://www.mat.puc-rio.br/~nicolau/olimp/obm-l.html
=


Re: [obm-l] Questao da Eureka 01

2004-04-27 Por tôpico Fellipe Rossi



bom o produto das razes  -3/a e a soma 
-b/a
uma delas  -1
logo -1xp=-3/a - p=3/a.
p-1 = -b/a
3-a = -b
a-b = 3

Primos: {2,3,5,7,11,13,17,19,...}

Como a e b sao primos com diferena=3, a=5 e 
b=2 (nao precisamos pensar em numeros grandes visto que o maior valor do 
enunciado eh 85 e 11^2 ja seria maior do que isto, porm no sei como provar que 
os nicos primos em q a diferena vale 3 sero 2 e 5)

logo, a^2 + b^2 = 29.

Acho que  isso.

algum poderia dar uma olhada no parnteses? 
:)

Abraos,
Rossi

  - Original Message - 
  From: 
  [EMAIL PROTECTED] 
  
  To: [EMAIL PROTECTED] 
  Sent: Tuesday, April 27, 2004 7:04 
  PM
  Subject: [obm-l] Questao da Eureka 
  01
  Ola pessoal, Poderiam me explicar como se resolve esta: 
  1) A equacao do 2 grau ax^2 + bx  3 = 0 tem 1 como uma de 
  suasraizes. Sabendo que os coeficientes a e b sao numeros primos 
  positivos, podemos afirmar que a^2 + b^2 eh igual a:a) 29 b) 89 c) 17 
  d) 13 e) 53Ps: Alguem poderia me enviar a figura a que se refere a 
  questao de treinamento (numero 02) da eureka 01 ? Fiz o download da revista, 
  mas nao aparece esta figura. 


Re: [obm-l] Questao da Eureka 01 - Desculpem!!

2004-04-27 Por tôpico Fellipe Rossi
Title: Re: [obm-l] Questao da Eureka 01



Ops!!
Eram tantas msgns que acabei deixando essa passar e 
mandei denovo a mesma coisa

Sorry =/

Rossi

  - Original Message - 
  From: 
  Claudio Buffara 
  To: [EMAIL PROTECTED] 
  Sent: Tuesday, April 27, 2004 9:44 
  PM
  Subject: Re: [obm-l] Questao da Eureka 
  01
  on 27.04.04 19:04, [EMAIL PROTECTED] at [EMAIL PROTECTED] wrote:
  Ola pessoal, Poderiam me explicar como se 
resolve esta: 1) A equacao do 2º grau ax^2 + bx  3 = 0 
tem 1 como uma de suasraizes. Sabendo que os coeficientes a e b sao 
numeros primos positivos, podemos afirmar que a^2 + b^2 eh igual 
a:a) 29 b) 89 c) 17 d) 13 e) 53as raizes sao -1 a u == 
u - 1 = -b/a e -u = -3/a ==3/a - 1 = -b/a 
==3 - a = -b ==a = b + 3 ==a  b e a, 
b tem paridades distintas.Como eles sao primos positivos, soh pode 
ser b = 2 e a = 5 ==a^2 + b^2 = 25 + 4 = 
29[]s,Claudio.


Re: [obm-l] Questao da Eureka 01

2004-04-27 Por tôpico 234



Se a diferena entre dois primos  3, ento um  
par, outro mpar.

A  automtico que um deles  2...

234

  - Original Message - 
  From: 
  Fellipe Rossi 
  To: [EMAIL PROTECTED] 
  Sent: Tuesday, April 27, 2004 11:52 
  PM
  Subject: Re: [obm-l] Questao da Eureka 
  01
  
  bom o produto das razes  -3/a e a soma 
  -b/a
  uma delas  -1
  logo -1xp=-3/a - p=3/a.
  p-1 = -b/a
  3-a = -b
  a-b = 3
  
  Primos: {2,3,5,7,11,13,17,19,...}
  
  Como a e b sao primos com diferena=3, a=5 e 
  b=2 (nao precisamos pensar em numeros grandes visto que o maior valor do 
  enunciado eh 85 e 11^2 ja seria maior do que isto, porm no sei como provar 
  que os nicos primos em q a diferena vale 3 sero 2 e 5)
  
  logo, a^2 + b^2 = 29.
  
  Acho que  isso.
  
  algum poderia dar uma olhada no parnteses? 
  :)
  
  Abraos,
  Rossi
  
- Original Message - 
From: 
[EMAIL PROTECTED] 
To: [EMAIL PROTECTED] 
Sent: Tuesday, April 27, 2004 7:04 
PM
Subject: [obm-l] Questao da Eureka 
01
Ola pessoal, Poderiam me explicar como se resolve esta: 
1) A equacao do 2 grau ax^2 + bx  3 = 0 tem 1 como uma de 
suasraizes. Sabendo que os coeficientes a e b sao numeros primos 
positivos, podemos afirmar que a^2 + b^2 eh igual a:a) 29 b) 89 c) 
17 d) 13 e) 53Ps: Alguem poderia me enviar a figura a que se refere 
a questao de treinamento (numero 02) da eureka 01 ? Fiz o download da 
revista, mas nao aparece esta figura. 



[obm-l] Problema legal

2004-04-27 Por tôpico Marcelo Souza
Olá pessoas 

Alguém poderia me dar uma mãozinha neste probleminha

1. João resolve equações quadráticas. Resolvendo a equação x^2+p_1x+q_1=0, ele encontra duas raízes reais p_2, q_2, p_2q_2. Então ele resolve x^2+p_2x+q_2=0 e assim por diante...
Até quando este exercício se repetirá, sabendo que João não conhece números complexos?

obrigado
[]'s, Marcelo.MSN Messenger: converse com os seus amigos online. Instale grátis. Clique aqui. 
=
Instruções para entrar na lista, sair da lista e usar a lista em
http://www.mat.puc-rio.br/~nicolau/olimp/obm-l.html
=


RE: [obm-l] Binomios... Duvida ( interessante )

2004-04-27 Por tôpico Rogério Moraes de Carvalho








Olá Fábio,



Segue uma solução possível
para este problema.



Sejam a[k] e b[p] os
termos gerais dos binômios de Newton (2x - 3)^4 e (x + 2)^5, respectivamente,
termos:

a[k] = BINOM(4, k).(2x)^k.(-3)^(4
- k), com k pertencente a {0, 1, 2, 3, 4}

b[p] = BINOM(5, p).x^p.2^(5
- p), com k pertencente a {0, 1, 2, 3, 4}



Na multiplicação das
potências dos binômios, teremos que todos os termos a[k] possíveis serão
multiplicados por todos os termos b[p] possíveis, por aplicação da propriedade
distributiva.

Portanto:

a[k].b[p] = BINOM(4, k).BINOM(5,
p).2^k.(-3)^(4 - k).2^(5 - p).x^(k + p)



Devemos encontrar todos
os pares (k, p) tais que k + p = 3: (0, 3), (1, 2), (2, 1) e (3, 0).



Para k = 0 e p = 3: BINOM(4,
0).BINOM(5, 3).2^0.(-3)^4.2^2.x^3 = 3240.x^3

Para k = 1 e p = 2: BINOM(4,
1).BINOM(5, 2).2^1.(-3)^3.2^3.x^3 = -17280.x^3

Para k = 2 e p = 1: BINOM(4,
2).BINOM(5, 1).2^2.(-3)^2.2^4.x^3 = 17280.x^3

Para k = 3 e p = 0: BINOM(4,
3).BINOM(5, 0).2^3.(-3)^1.2^5.x^3 = -3072.x^3



Adicionando os termos: 3240.x^3
+ (-17280.x^3) + 17280.x^3 + (-3072.x^3) = 168.x^3



Portanto, o coeficiente
de x^3 é igual a 168.



Abraços,





Rogério Moraes de Carvalho

Consultor
e Instrutor de Tecnologias da Informação

[EMAIL PROTECTED]











From: owner-[EMAIL PROTECTED] [mailto:owner-[EMAIL PROTECTED]] On Behalf Of Fabio Contreiras
Sent: terça-feira, 27 de abril de
2004 22:45
To: [EMAIL PROTECTED]
Subject: [obm-l] Binomios...
Duvida ( interessante )







1) Determinar o coeficiente de x^3 no desenvolvimento de (
2x - 3 )^4 . ( x + 2 )^5

















Alguem pode me explicar o caminho ?





Abração!










Re: [obm-l] Integral...

2004-04-27 Por tôpico Rafael



Uma outra sugestão é utilizar substituições da 
trigonometria hiperbólica. 

Você chegariaà expressão:sqrt(2) atanh[(tan(x/2) - 1)/sqrt(2)]


[]s,

Rafael



  - Original Message - 
  From: 
  Alan Pellejero 
  To: [EMAIL PROTECTED] 
  Sent: Tuesday, April 27, 2004 5:36 
  PM
  Subject: [obm-l] Integral...
  
  Olá amigos da lista, 
  pessoal, gostaria de saber se alguém tem uma "carta na manga para esse 
  aqui..."
  
  /|1/ (senx + cosx) dx 
  |
  /
  
  Eu fiz de uma maneira "corinthiana"...ou seja, deu 2 folhas!!!
  Queria saber se alguém tem uma solução são-paulina (inteligente, rápida, 
  objetiva, concisa...)
  
  Ps: esse não é o da hipociclóide de novo.rs
  valeu!
  té mais!
  Alan Pellejero